Difference between revisions of "1985 AJHSME Problems/Problem 21"

m
(Solution)
(14 intermediate revisions by 6 users not shown)
Line 7: Line 7:
 
==Solution==
 
==Solution==
  
If he gets a <math>10\% </math> raise, then his new salary is <math>1.1</math> of what it was before. He gets <math>4</math> raises, so his new salary is <math>1.1^4</math> of the previous salary. We want to know the percent of increase from <math>1</math>, so we just need <math>1.1^4-1</math> as a percent.
+
Assume his salary is originally <math>100</math> dollars. Then, in the next year, he would have <math>110</math> dollars, and in the next, he would have <math>121</math> dollars. The next year he would have <math>133.1</math> dollars and in the final year, he would have <math>146.41</math>. As the total increase is greater than <math>45\%</math>, the answer is <math>\boxed{\text{E}}</math>.  
  
If you know your powers of <math>11</math> well, then you can just do <math>1.4641-1=.4641>45\% </math>, so the answer is <math>\boxed{\text{E}}</math>
+
~sakshamsethi
 
 
If you don't know this, then first we note that <math>1.1^4</math> clearly does not have a whole number percent representation, so the only possible choices are <math>\text{A}</math> and <math>\text{E}</math>. We also have <cmath>1.1^4=1.21^2>1.2^2=1.44</cmath>
 
 
 
so the percent is greater than <math>40\% </math> and the only choice left is <math>\boxed{\text{E}}</math>.
 
  
 
==See Also==
 
==See Also==
Line 19: Line 15:
 
{{AJHSME box|year=1985|num-b=20|num-a=22}}
 
{{AJHSME box|year=1985|num-b=20|num-a=22}}
 
[[Category:Introductory Algebra Problems]]
 
[[Category:Introductory Algebra Problems]]
 +
 +
 +
{{MAA Notice}}

Revision as of 20:16, 26 September 2020

Problem

Mr. Green receives a $10\%$ raise every year. His salary after four such raises has gone up by what percent?

$\text{(A)}\ \text{less than }40\% \qquad \text{(B)}\ 40\% \qquad \text{(C)}\ 44\% \qquad \text{(D)}\ 45\% \qquad \text{(E)}\ \text{more than }45\%$

Solution

Assume his salary is originally $100$ dollars. Then, in the next year, he would have $110$ dollars, and in the next, he would have $121$ dollars. The next year he would have $133.1$ dollars and in the final year, he would have $146.41$. As the total increase is greater than $45\%$, the answer is $\boxed{\text{E}}$.

~sakshamsethi

See Also

1985 AJHSME (ProblemsAnswer KeyResources)
Preceded by
Problem 20
Followed by
Problem 22
1 2 3 4 5 6 7 8 9 10 11 12 13 14 15 16 17 18 19 20 21 22 23 24 25
All AJHSME/AMC 8 Problems and Solutions


The problems on this page are copyrighted by the Mathematical Association of America's American Mathematics Competitions. AMC logo.png